Question and Answers Forum

All Questions      Topic List

Permutation and Combination Questions

Previous in All Question      Next in All Question      

Previous in Permutation and Combination      Next in Permutation and Combination      

Question Number 56913 by Joel578 last updated on 26/Mar/19

How many possible solution sets that satisfy   x_1  + x_2  + x_3  + x_4  = 5  with  0 ≤ x_1  ≤ 3    0 ≤ x_2  ≤ 3  0 ≤ x_3  ≤ 2  0 ≤ x_4  ≤ 2

$$\mathrm{How}\:\mathrm{many}\:\mathrm{possible}\:\mathrm{solution}\:\mathrm{sets}\:\mathrm{that}\:\mathrm{satisfy}\: \\ $$$${x}_{\mathrm{1}} \:+\:{x}_{\mathrm{2}} \:+\:{x}_{\mathrm{3}} \:+\:{x}_{\mathrm{4}} \:=\:\mathrm{5} \\ $$$$\mathrm{with} \\ $$$$\mathrm{0}\:\leqslant\:{x}_{\mathrm{1}} \:\leqslant\:\mathrm{3}\:\: \\ $$$$\mathrm{0}\:\leqslant\:{x}_{\mathrm{2}} \:\leqslant\:\mathrm{3} \\ $$$$\mathrm{0}\:\leqslant\:{x}_{\mathrm{3}} \:\leqslant\:\mathrm{2} \\ $$$$\mathrm{0}\:\leqslant\:{x}_{\mathrm{4}} \:\leqslant\:\mathrm{2} \\ $$

Commented by 121194 last updated on 26/Mar/19

3 2 0 0  3 1 1 0  3 1 0 1  3 0 2 0  3 0 1 1  3 0 0 2  2 3 0 0  2 2 1 0  2 2 0 1  2 1 2 0  2 1 1 1  2 1 0 2  2 0 2 1  2 0 1 2  1 3 1 0  1 3 0 1  1 2 2 0  1 2 1 1  1 2 0 2  1 1 2 1  1 1 1 2  1 0 2 2  0 3 2 0  0 3 1 1  0 3 0 2  0 2 2 1  0 2 1 2  0 1 2 2  28

$$\mathrm{3}\:\mathrm{2}\:\mathrm{0}\:\mathrm{0} \\ $$$$\mathrm{3}\:\mathrm{1}\:\mathrm{1}\:\mathrm{0} \\ $$$$\mathrm{3}\:\mathrm{1}\:\mathrm{0}\:\mathrm{1} \\ $$$$\mathrm{3}\:\mathrm{0}\:\mathrm{2}\:\mathrm{0} \\ $$$$\mathrm{3}\:\mathrm{0}\:\mathrm{1}\:\mathrm{1} \\ $$$$\mathrm{3}\:\mathrm{0}\:\mathrm{0}\:\mathrm{2} \\ $$$$\mathrm{2}\:\mathrm{3}\:\mathrm{0}\:\mathrm{0} \\ $$$$\mathrm{2}\:\mathrm{2}\:\mathrm{1}\:\mathrm{0} \\ $$$$\mathrm{2}\:\mathrm{2}\:\mathrm{0}\:\mathrm{1} \\ $$$$\mathrm{2}\:\mathrm{1}\:\mathrm{2}\:\mathrm{0} \\ $$$$\mathrm{2}\:\mathrm{1}\:\mathrm{1}\:\mathrm{1} \\ $$$$\mathrm{2}\:\mathrm{1}\:\mathrm{0}\:\mathrm{2} \\ $$$$\mathrm{2}\:\mathrm{0}\:\mathrm{2}\:\mathrm{1} \\ $$$$\mathrm{2}\:\mathrm{0}\:\mathrm{1}\:\mathrm{2} \\ $$$$\mathrm{1}\:\mathrm{3}\:\mathrm{1}\:\mathrm{0} \\ $$$$\mathrm{1}\:\mathrm{3}\:\mathrm{0}\:\mathrm{1} \\ $$$$\mathrm{1}\:\mathrm{2}\:\mathrm{2}\:\mathrm{0} \\ $$$$\mathrm{1}\:\mathrm{2}\:\mathrm{1}\:\mathrm{1} \\ $$$$\mathrm{1}\:\mathrm{2}\:\mathrm{0}\:\mathrm{2} \\ $$$$\mathrm{1}\:\mathrm{1}\:\mathrm{2}\:\mathrm{1} \\ $$$$\mathrm{1}\:\mathrm{1}\:\mathrm{1}\:\mathrm{2} \\ $$$$\mathrm{1}\:\mathrm{0}\:\mathrm{2}\:\mathrm{2} \\ $$$$\mathrm{0}\:\mathrm{3}\:\mathrm{2}\:\mathrm{0} \\ $$$$\mathrm{0}\:\mathrm{3}\:\mathrm{1}\:\mathrm{1} \\ $$$$\mathrm{0}\:\mathrm{3}\:\mathrm{0}\:\mathrm{2} \\ $$$$\mathrm{0}\:\mathrm{2}\:\mathrm{2}\:\mathrm{1} \\ $$$$\mathrm{0}\:\mathrm{2}\:\mathrm{1}\:\mathrm{2} \\ $$$$\mathrm{0}\:\mathrm{1}\:\mathrm{2}\:\mathrm{2} \\ $$$$\mathrm{28} \\ $$

Commented by Joel578 last updated on 27/Mar/19

thank you very much

$${thank}\:{you}\:{very}\:{much} \\ $$

Answered by mr W last updated on 26/Mar/19

using generating functions  x_1 : 1+t+t^2 +t^3   x_2 : 1+t+t^2 +t^3   x_3 : 1+t+t^2   x_4 : 1+t+t^2   x_1 +x_2 +x_3 +x_4 : (1+t+t^2 +t^3 )^2 (1+t+t^2 )^2 =(((1−t^4 )^2 (1−t^3 )^2 )/((1−t)^4 ))  =Σ_(k=0) ^2 C_k ^2 (−1)^k t^(4k) Σ_(k=0) ^2 C_k ^2 (−1)^k t^(3k) Σ_(k=0) ^∞ C_k ^(3+k) t^k     coef. of term t^5 :  C_1 ^2 (−1)^1 C_0 ^2 (−1)^0 C_1 ^4 +C_0 ^2 (−1)^0 C_1 ^2 (−1)^1 C_2 ^5 +C_0 ^2 (−1)^0 C_0 ^2 (−1)^0 C_5 ^8   =−C_1 ^2 C_0 ^2 C_1 ^4 −C_0 ^2 C_1 ^2 C_2 ^5 +C_0 ^2 C_0 ^2 C_5 ^8   =−2×4−2×10+56  =28    ⇒there are 28 possible solutions for  x_1  + x_2  + x_3  + x_4  = 5  under given conditions.    other terms see below:

$${using}\:{generating}\:{functions} \\ $$$${x}_{\mathrm{1}} :\:\mathrm{1}+{t}+{t}^{\mathrm{2}} +{t}^{\mathrm{3}} \\ $$$${x}_{\mathrm{2}} :\:\mathrm{1}+{t}+{t}^{\mathrm{2}} +{t}^{\mathrm{3}} \\ $$$${x}_{\mathrm{3}} :\:\mathrm{1}+{t}+{t}^{\mathrm{2}} \\ $$$${x}_{\mathrm{4}} :\:\mathrm{1}+{t}+{t}^{\mathrm{2}} \\ $$$${x}_{\mathrm{1}} +{x}_{\mathrm{2}} +{x}_{\mathrm{3}} +{x}_{\mathrm{4}} :\:\left(\mathrm{1}+{t}+{t}^{\mathrm{2}} +{t}^{\mathrm{3}} \right)^{\mathrm{2}} \left(\mathrm{1}+{t}+{t}^{\mathrm{2}} \right)^{\mathrm{2}} =\frac{\left(\mathrm{1}−{t}^{\mathrm{4}} \right)^{\mathrm{2}} \left(\mathrm{1}−{t}^{\mathrm{3}} \right)^{\mathrm{2}} }{\left(\mathrm{1}−{t}\right)^{\mathrm{4}} } \\ $$$$=\underset{{k}=\mathrm{0}} {\overset{\mathrm{2}} {\sum}}{C}_{{k}} ^{\mathrm{2}} \left(−\mathrm{1}\right)^{{k}} {t}^{\mathrm{4}{k}} \underset{{k}=\mathrm{0}} {\overset{\mathrm{2}} {\sum}}{C}_{{k}} ^{\mathrm{2}} \left(−\mathrm{1}\right)^{{k}} {t}^{\mathrm{3}{k}} \underset{{k}=\mathrm{0}} {\overset{\infty} {\sum}}{C}_{{k}} ^{\mathrm{3}+{k}} {t}^{{k}} \\ $$$$ \\ $$$${coef}.\:{of}\:{term}\:{t}^{\mathrm{5}} : \\ $$$${C}_{\mathrm{1}} ^{\mathrm{2}} \left(−\mathrm{1}\right)^{\mathrm{1}} {C}_{\mathrm{0}} ^{\mathrm{2}} \left(−\mathrm{1}\right)^{\mathrm{0}} {C}_{\mathrm{1}} ^{\mathrm{4}} +{C}_{\mathrm{0}} ^{\mathrm{2}} \left(−\mathrm{1}\right)^{\mathrm{0}} {C}_{\mathrm{1}} ^{\mathrm{2}} \left(−\mathrm{1}\right)^{\mathrm{1}} {C}_{\mathrm{2}} ^{\mathrm{5}} +{C}_{\mathrm{0}} ^{\mathrm{2}} \left(−\mathrm{1}\right)^{\mathrm{0}} {C}_{\mathrm{0}} ^{\mathrm{2}} \left(−\mathrm{1}\right)^{\mathrm{0}} {C}_{\mathrm{5}} ^{\mathrm{8}} \\ $$$$=−{C}_{\mathrm{1}} ^{\mathrm{2}} {C}_{\mathrm{0}} ^{\mathrm{2}} {C}_{\mathrm{1}} ^{\mathrm{4}} −{C}_{\mathrm{0}} ^{\mathrm{2}} {C}_{\mathrm{1}} ^{\mathrm{2}} {C}_{\mathrm{2}} ^{\mathrm{5}} +{C}_{\mathrm{0}} ^{\mathrm{2}} {C}_{\mathrm{0}} ^{\mathrm{2}} {C}_{\mathrm{5}} ^{\mathrm{8}} \\ $$$$=−\mathrm{2}×\mathrm{4}−\mathrm{2}×\mathrm{10}+\mathrm{56} \\ $$$$=\mathrm{28} \\ $$$$ \\ $$$$\Rightarrow{there}\:{are}\:\mathrm{28}\:{possible}\:{solutions}\:{for} \\ $$$${x}_{\mathrm{1}} \:+\:{x}_{\mathrm{2}} \:+\:{x}_{\mathrm{3}} \:+\:{x}_{\mathrm{4}} \:=\:\mathrm{5} \\ $$$${under}\:{given}\:{conditions}. \\ $$$$ \\ $$$${other}\:{terms}\:{see}\:{below}: \\ $$

Commented by mr W last updated on 26/Mar/19

Commented by mr W last updated on 26/Mar/19

x_1 +x_2 +x_3 +x_4 =0 ⇒ 1 solution  x_1 +x_2 +x_3 +x_4 =1 ⇒ 4 solutions  x_1 +x_2 +x_3 +x_4 =2 ⇒ 10 solutions  x_1 +x_2 +x_3 +x_4 =3 ⇒ 18 solutions  x_1 +x_2 +x_3 +x_4 =4 ⇒ 25 solutions  x_1 +x_2 +x_3 +x_4 =5 ⇒ 28 solutions  x_1 +x_2 +x_3 +x_4 =6 ⇒ 25 solutions  x_1 +x_2 +x_3 +x_4 =7 ⇒ 18 solutions  x_1 +x_2 +x_3 +x_4 =8 ⇒ 10 solutions  x_1 +x_2 +x_3 +x_4 =9 ⇒ 4 solutions  x_1 +x_2 +x_3 +x_4 =10 ⇒ 1 solution

$${x}_{\mathrm{1}} +{x}_{\mathrm{2}} +{x}_{\mathrm{3}} +{x}_{\mathrm{4}} =\mathrm{0}\:\Rightarrow\:\mathrm{1}\:{solution} \\ $$$${x}_{\mathrm{1}} +{x}_{\mathrm{2}} +{x}_{\mathrm{3}} +{x}_{\mathrm{4}} =\mathrm{1}\:\Rightarrow\:\mathrm{4}\:{solutions} \\ $$$${x}_{\mathrm{1}} +{x}_{\mathrm{2}} +{x}_{\mathrm{3}} +{x}_{\mathrm{4}} =\mathrm{2}\:\Rightarrow\:\mathrm{10}\:{solutions} \\ $$$${x}_{\mathrm{1}} +{x}_{\mathrm{2}} +{x}_{\mathrm{3}} +{x}_{\mathrm{4}} =\mathrm{3}\:\Rightarrow\:\mathrm{18}\:{solutions} \\ $$$${x}_{\mathrm{1}} +{x}_{\mathrm{2}} +{x}_{\mathrm{3}} +{x}_{\mathrm{4}} =\mathrm{4}\:\Rightarrow\:\mathrm{25}\:{solutions} \\ $$$${x}_{\mathrm{1}} +{x}_{\mathrm{2}} +{x}_{\mathrm{3}} +{x}_{\mathrm{4}} =\mathrm{5}\:\Rightarrow\:\mathrm{28}\:{solutions} \\ $$$${x}_{\mathrm{1}} +{x}_{\mathrm{2}} +{x}_{\mathrm{3}} +{x}_{\mathrm{4}} =\mathrm{6}\:\Rightarrow\:\mathrm{25}\:{solutions} \\ $$$${x}_{\mathrm{1}} +{x}_{\mathrm{2}} +{x}_{\mathrm{3}} +{x}_{\mathrm{4}} =\mathrm{7}\:\Rightarrow\:\mathrm{18}\:{solutions} \\ $$$${x}_{\mathrm{1}} +{x}_{\mathrm{2}} +{x}_{\mathrm{3}} +{x}_{\mathrm{4}} =\mathrm{8}\:\Rightarrow\:\mathrm{10}\:{solutions} \\ $$$${x}_{\mathrm{1}} +{x}_{\mathrm{2}} +{x}_{\mathrm{3}} +{x}_{\mathrm{4}} =\mathrm{9}\:\Rightarrow\:\mathrm{4}\:{solutions} \\ $$$${x}_{\mathrm{1}} +{x}_{\mathrm{2}} +{x}_{\mathrm{3}} +{x}_{\mathrm{4}} =\mathrm{10}\:\Rightarrow\:\mathrm{1}\:{solution} \\ $$

Commented by Joel578 last updated on 27/Mar/19

thank you very much

$${thank}\:{you}\:{very}\:{much} \\ $$

Terms of Service

Privacy Policy

Contact: info@tinkutara.com